- Fri Jan 21, 2011 12:00 am
#27101
Complete Question Explanation
(The complete setup for this game can be found here: lsat/viewtopic.php?t=11371)
The correct answer choice is (C)
Answer choice (C) is the correct answer choice because U and X could both repair R and T, or R and V. In each of the other four answer choices the pairs can never repair the exact same types (or numbers) of machines:
Answer choice (A) is incorrect because S repairs one machine and Y must repair two machines.
Answer choice (B) is incorrect because although U and Y can repair the same number of machines, U repairs R whereas Y does not.
Answer choice (D) is incorrect because although W and X can repair the same number of machines, X repairs R whereas W does not.
(The complete setup for this game can be found here: lsat/viewtopic.php?t=11371)
The correct answer choice is (C)
Answer choice (C) is the correct answer choice because U and X could both repair R and T, or R and V. In each of the other four answer choices the pairs can never repair the exact same types (or numbers) of machines:
Answer choice (A) is incorrect because S repairs one machine and Y must repair two machines.
Answer choice (B) is incorrect because although U and Y can repair the same number of machines, U repairs R whereas Y does not.
Answer choice (D) is incorrect because although W and X can repair the same number of machines, X repairs R whereas W does not.